Đến nội dung

Hình ảnh

Đề thi thử THPT chuyên KHTN Lớp 9 Vòng 2 - Đợt 4 Năm 2017

khtn

  • Please log in to reply
Chủ đề này có 9 trả lời

#1
Mr Cooper

Mr Cooper

    Sĩ quan

  • Thành viên
  • 496 Bài viết

18425229_1703317736350060_52130823470118



#2
tienduc

tienduc

    Thiếu úy

  • Điều hành viên THCS
  • 580 Bài viết

TRƯỜNG ĐẠI HỌC KHOA HỌC TỰ NHIÊN                                          ĐỀ THI THỬ LỚP 9 NĂM 2017

      TRƯỜNG THPT CHUYÊN KHTN                                                      Môn: Toán (Vòng 2 - Đợt 4)

                                                                                                 Thời gian: $150$ phút (không kể thời gian phát đề)

 

 

Câu I (3 điểm)

1)  Giải hệ phương trình            $\left\{\begin{matrix} 2x^2+y^2+3xy=6 & \\ y^2+2xy+2x+y=6 & \end{matrix}\right.$

2) Chứng minh rằng biểu thức sau 

$M=(\frac{1+ab}{1-ab})(\frac{b+c}{b-c})+(\frac{b+c}{b-c})(\frac{ac+1}{ac-1})+(\frac{ac+1}{ac-1})(\frac{1+ab}{1-ab})$

là số nguyên với mọi bộ số thực $a,b,c$

 

Câu II (3 điểm)

1) Tìm $x,y$ nguyên thỏa mãn $(y+2)(x^2+1)=2x^3+3x+1$

2) Có bao nhiêu cách xếp $10$ người có chiều cao đôi một phân biệt thành một hàng ngang sao cho trong hàng mọi người luôn cao hơn hoặc luôn thấp hơn tất cả các người xếp trước. 

 

Câu III (3 điểm)

Cho đường tròn $(O)$ có dây cung $AB$ không là đường kính. $C$ là một điểm thuộc đoạn $AB$ và $D$ là một điểm nằm trên cung nhỏ $\overrightarrow{AB}$ của $(O)$. $OD$ cắt $AB$ tại $E$. $OC$ cắt đường tròn $(K)$ ngoại tiếp tam giác $OAB$ tại $F$ khác $O$

1) Chứng minh rằng $\angle CFD = \angle CDO$

2) Gọi $FE$ cắt $(K)$ tại $G$ khác $F$. $GD$ cắt $(K)$ tại $H$ khác $G$. Chứng minh $OH$ chia đôi $CD$

 

Câu IV(1 điểm) Cho $a,b,c$ là các số thực dương thỏa mãn $ab+bc+ca=1$. Chứng minh 

$ \sum \frac{1}{2+a^2} \leq \frac{9}{7}$



#3
Hoang Dinh Nhat

Hoang Dinh Nhat

    Sĩ quan

  • Thành viên
  • 402 Bài viết

Câu II (3 điểm)

1) Tìm $x,y$ nguyên thỏa mãn $(y+2)(x^2+1)=2x^3+3x+1$

 

 

phương trình $\Leftrightarrow \frac{2x^3-2x^2+3x-1}{x^2+1}=2(x-1)+\frac{x}{x^2+1}+\frac{1}{x^2+1}$ (1)

Để phương trình có nghiệm nguyên thì $1\vdots x^2+1$  ;$x\vdots x^2+1$(*)

Đầu tiên để có nghiệm nguyên thì $x^2+1$ là ước của 1

TH1: $x^2+1=1\Leftrightarrow x^2=0\Leftrightarrow x=0$. Thay vào (*) ta thấy thỏa mãn.

TH2:$x^2+1=-1\Leftrightarrow x^2=-2$ (vô lý)

Vì vậy chỉ có $x=0$ là giá trị nguyên thỏa mãn. THay x=0 vào (1) ta được $y=-1$

Vậy phương trình có một nghiệm nguyên: $(x;y)=(0;-1)$


Bài viết đã được chỉnh sửa nội dung bởi Hoang Dinh Nhat: 14-05-2017 - 13:30

Chấp nhận giới hạn của bản thân, nhưng đừng bao giờ bỏ cuộc

 

 

 

 


#4
viet9a14124869

viet9a14124869

    Trung úy

  • Thành viên
  • 903 Bài viết

18425229_1703317736350060_52130823470118

Chém câu bất dễ nhất

Đẳng thức cần chứng minh tương đương $(1-\frac{2}{2+a^2})+(1-\frac{2}{2+b^2})+(1-\frac{2}{2+c^2})\geq \frac{3}{7}$

Tới đây thì dùng Schwarz , $\Rightarrow \sum \frac{a^2}{a^2+2}\geq \frac{(a+b+c)^2}{a^2+b^2+c^2+6}=\frac{(a+b+c)^2}{a^2+b^2+c^@+6ab+6bc+6ca}\geq \frac{3}{7}\Leftrightarrow a^2+b^2+c^2\geq ab+bc+ca$  ( đúng theo AM-GM )

Vậy bất đẳng thức được chứng minh :D

Dấu bằng xảy ra khi $a=b=c=\frac{1}{\sqrt{3}}$ !


Bài viết đã được chỉnh sửa nội dung bởi viet9a14124869: 14-05-2017 - 14:19

                                                                    SÓNG BẮT ĐẦU TỪ GIÓ

                                                                    GIÓ BẮT ĐẦU TỪ ĐÂU ?

                                                                    ANH CŨNG KHÔNG BIẾT NỮA 

                                                                    KHI NÀO...? TA YÊU NHAU .


#5
Hoang Dinh Nhat

Hoang Dinh Nhat

    Sĩ quan

  • Thành viên
  • 402 Bài viết

 

$M=(\frac{1+ab}{1-ab})(\frac{b+c}{b-c})+(\frac{b+c}{b-c})(\frac{ac+1}{ac-1})+(\frac{ac+1}{ac-1})(\frac{1+ab}{1-ab})$

là số nguyên với mọi bộ số thực $a,b,c$

 

 

Biến đổi về $M=\frac{(ab-1)(ac-1)(b-c)}{(1-ab)(ac-1)(b-c)}=-1$ với mọi bộ số thực a,b,c thỏa mãn $b\neq c$


Bài viết đã được chỉnh sửa nội dung bởi Hoang Dinh Nhat: 14-05-2017 - 14:00

Chấp nhận giới hạn của bản thân, nhưng đừng bao giờ bỏ cuộc

 

 

 

 


#6
Mr Cooper

Mr Cooper

    Sĩ quan

  • Thành viên
  • 496 Bài viết

Câu I (3 điểm)

1)  Giải hệ phương trình            $\left\{\begin{matrix} 2x^2+y^2+3xy=6 & \\ y^2+2xy+2x+y=6 & \end{matrix}\right.$

Câu I. 1)Trừ $\text{PT}$ $(1)$ và $\text{PT}$ $(2)$ theo vế $\Leftrightarrow (2x+y)(x-1)=0$


Bài viết đã được chỉnh sửa nội dung bởi Mr Cooper: 14-05-2017 - 14:02


#7
Hoang Dinh Nhat

Hoang Dinh Nhat

    Sĩ quan

  • Thành viên
  • 402 Bài viết

Chém câu bất dễ nhất

Đẳng thức cần chứng minh tương đương $(1-\frac{2}{a^2+2})+(1-\frac{2}{2+b^2})+(1-\frac{2}{2+c^2})\geq \frac{5}{7}$

Tới đây thì dùng Schwarz , $\Rightarrow \sum \frac{a^2}{a^2+2}\geq \frac{(a+b+c)^2}{a^2+b^2+c^2+6}=\frac{(a+b+c)^2}{a^2+b^2+c^2+6ab+6bc+6ca}\geq \frac{5}{7}\Leftrightarrow a^2+b^2+c^2\geq ab+bc+ca$ ( đúng theo AM-GM )

Vậy bất đẳng thức được chứng minh :D

Dấu bằng xảy ra khi $a=b=c=\frac{1}{\sqrt{3}}$ !

BĐT cần chứng minh$\Leftrightarrow (1-\frac{2}{a^2+2})+(1-\frac{2}{2+b^2})+(1-\frac{2}{2+c^2})\geq \frac{3}{7}$ chứ nhỉ


Chấp nhận giới hạn của bản thân, nhưng đừng bao giờ bỏ cuộc

 

 

 

 


#8
Mr Cooper

Mr Cooper

    Sĩ quan

  • Thành viên
  • 496 Bài viết

Lời giải bài Hình của thầy Nguyễn Lê Phước

 

18425146_1703320586349775_12530876647399


Bài viết đã được chỉnh sửa nội dung bởi Mr Cooper: 14-05-2017 - 14:51


#9
Minhnksc

Minhnksc

    Sĩ quan

  • Điều hành viên OLYMPIC
  • 302 Bài viết

Còn câu nữa xử nốt cho nó lành

Câu II 

2)Gọi $X_{n}$ là số cách sắp xếp n người ($n\geq 0$) thỏa mãn đề bài (coi như $X_{0}=1$ vì khi không có người nào đứng thì chỉ có một cách sắp xếp duy nhất :) )

Giả sử chiều cao của $n$ người trên lần lượt là $a_{1};a_{2};...;a_{n}$ sao cho với $i>j$ thì $a_{i}>a_{j}$

Khi đó; giả sử người có chiều cao là $a_{1}$ đứng ở vị trí thứ $k$($0\leq k\leq n$) thì mỗi người đứng ở vị trí từ $1\rightarrow k$ luôn cao hơn tất cả những người đứng trước vì nếu ngược lại thì tồn tại một người có chiều cao nhỏ hơn $a_{1}$ (vô lý) $\Rightarrow$ người đứng ở vị trí thứ 1 là người cao nhất và có chiều cao $a_{n}$

Tương tự; người đứng ở vị trí thứ 2 có chiều cao là $a_{n-1}$

                 người đứng ở vị trí thứ 3 có chiều cao là $a_{n-2}$

                 .............

                 người đứng ở vị trí thứ $k-1$ có chiều cao là $a_{n-k}$

Do đó với mọi $0\leq k\leq n$ thì chỉ có một cách sắp sếp những người đứng ở vị trí từ 1 đến k sao cho thỏa mãn đề bài. Từ đó suy ra số cách sắp xếp n người thỏa mãn đề bài khi người thấp nhất (có chiều cao $a_{1}$) đứng ở vị trí thứ $k$ chính bằng số cách sắp xếp $n-k$ người đứng từ vị trí thứ $k+1$ trở đi ($=X_{n-k}$).

Vì vậy; khi cho $k$ lần lượt bằng $n;n-1;...;1$ thì ta nhận được số cách sắp xếp $n$ người lần lượt là $X_{0};X_{1};X_{2};...;X_{n-1}$

Mà tổng số cách sắp xếp $n$ người bằng tổng các số trên nên $X_{n}=X_{0}+X_{1}+...+X_{n-1}$

Lại có $X_{0}=1$ và $X_{1}=1$ nên từ công thức trên và bằng phương pháp quy nạp ta chứng minh được $X_{n}=2^{n-1} (n\geq 1)$

Thay $n= 10$ vào trên ta có $X_{10}=512$

P/S: Mới off có sáng với chiều chủ nhật để đi chơi mà tối lên diễn đàn đã thấy đề KHTN bị "xơi" gần hết rồi; còn mỗi cấu tổ :))


Bài viết đã được chỉnh sửa nội dung bởi Minhnksc: 16-05-2017 - 10:54

Sống khỏe và sống tốt :D


#10
Uchiha sisui

Uchiha sisui

    Trung sĩ

  • Thành viên
  • 196 Bài viết

Còn câu nữa xử nốt cho nó lành

Câu II 

2)Gọi $X_{n}$ là số cách sắp xếp n người ($n\geq 0$) thỏa mãn đề bài (coi như $X_{0}=1$ vì khi không có người nào đứng thì chỉ có một cách sắp xếp duy nhất :) )

Giả sử chiều cao của $n$ người trên lần lượt là $a_{1};a_{2};...;a_{n}$ sao cho với $i>j$ thì $a_{i}>a_{j}$

Khi đó; giả sử người có chiều cao là $a_{1}$ đứng ở vị trí thứ $k$($0\leq k\leq n$) thì mỗi người đứng ở vị trí từ $1\rightarrow k$ luôn cao hơn tất cả những người đứng trước vì nếu ngược lại thì tồn tại một người có chiều cao nhỏ hơn $a_{1}$ (vô lý) $\Rightarrow$ người đứng ở vị trí thứ 1 là người cao nhất và có chiều cao $a_{n}$

Tương tự; người đứng ở vị trí thứ 2 có chiều cao là $a_{n-1}$

                 người đứng ở vị trí thứ 3 có chiều cao là $a_{n-2}$

                 .............

                 người đứng ở vị trí thứ $k-1$ có chiều cao là $a_{n-k}$

Do đó với mọi $0\leq k\leq n$ thì chỉ có một cách sắp sếp những người đứng ở vị trí từ 1 đến k sao cho thỏa mãn đề bài. Từ đó suy ra số cách sắp xếp n người thỏa mãn đề bài khi người thấp nhất (có chiều cao $a_{1}$) đứng ở vị trí thứ $k$ chính bằng số cách sắp xếp $n-k$ người đứng từ vị trí thứ $k+1$ trở đi ($=X_{n-k}$).

Vì vậy; khi cho $k$ lần lượt bằng $n;n-1;...;1$ thì ta nhận được số cách sắp xếp $n$ người lần lượt là $X_{0};X_{1};X_{2};...;X_{n-1}$

Mà tổng số cách sắp xếp $n$ người bằng tổng các số trên nên $X_{n}=X_{0}+X_{1}+...+X_{n-1}$

Lại có $X_{0}=1$ và $X_{1}=1$ nên từ công thức trên và bằng phương pháp quy nạp ta chứng minh được $X_{n}=2^{n-1} (n\geq 1)$

Thay $n= 10$ vào trên ta có $X_{10}=512$

P/S: Mới off có sáng với chiều chủ nhật để đi chơi mà tối lên diễn đàn đã thấy đề KHTN bị "xơi" gần hết rồi; còn mỗi cấu tổ :))

bác giỏi ghê :v trâu quá :V







Được gắn nhãn với một hoặc nhiều trong số những từ khóa sau: khtn

2 người đang xem chủ đề

0 thành viên, 2 khách, 0 thành viên ẩn danh